Is My Matrix Approach to Solving Linear Equations with Parameters Correct?

Click For Summary
The discussion revolves around solving a system of linear equations with parameters using a matrix approach and Gaussian elimination. The user is uncertain whether the presence of the parameter t in their solutions is correct, as they have seen similar problems where the solutions do not include parameters. It is clarified that for each specific value of t, the equations yield different solutions, which is why t appears in the results. However, for t=0, the equations conflict, leading to no solution for x, y, and z. There is also a note about a potential error in the notation used for the denominator in the user's equations.
monsmatglad
Messages
75
Reaction score
0

Homework Statement


hi. i have a linear equation with parameters that i am trying to solve. unknowns are x, y and z.
(12-3t)x + 2y +2z =2
x +0*y +2z =0
12x + (2-t)y + (2-t)z = 12

i am using a matrix-approach with Gaussian-elimination, and my results (for values of t which allows for a specific solution) seem reasonable, but the the variables depend on t. is this correct, or will the answers be without the parameter t even if t is not specified. i believe i have seen a similar problem in one of my books where the solution is without the parameter t.

Homework Equations


am i getting it wrong?

The Attempt at a Solution


x= -(2t+20)/(3t2t), y= -(37t-110)/(3t2t), z= (t+10)/(3t2t)
 
Last edited by a moderator:
Physics news on Phys.org
I think you're doing just fine. For every value of t (*) you have a different set of equations, so it's no wonder t appears in the solution.

(*) for t=0 you get conflicting equations, so it's not surprising there is no answer for x,y,z.
 
I am puzzled as to why you are writing "t^2t" rather than "t^3". Am I misunderstanding what you mean?
 
monsmatglad said:

The Attempt at a Solution


x= -(2t+20)/(3t2t), y= -(37t-110)/(3t2t), z= (t+10)/(3t2t)
Check the denominator. I think you just copied it wrong.
 
Question: A clock's minute hand has length 4 and its hour hand has length 3. What is the distance between the tips at the moment when it is increasing most rapidly?(Putnam Exam Question) Answer: Making assumption that both the hands moves at constant angular velocities, the answer is ## \sqrt{7} .## But don't you think this assumption is somewhat doubtful and wrong?

Similar threads

Replies
2
Views
1K
  • · Replies 7 ·
Replies
7
Views
2K
  • · Replies 3 ·
Replies
3
Views
1K
  • · Replies 3 ·
Replies
3
Views
2K
  • · Replies 1 ·
Replies
1
Views
2K
Replies
8
Views
3K
  • · Replies 4 ·
Replies
4
Views
1K
  • · Replies 2 ·
Replies
2
Views
945
Replies
7
Views
2K
  • · Replies 2 ·
Replies
2
Views
2K